Double Integration Homework: Changing Limits

  • Thread starter Thread starter jaus tail
  • Start date Start date
  • Tags Tags
    Integration
Click For Summary
The discussion focuses on a double integration homework problem involving changing limits of integration. The original limits are from y = 0 to y = x - 2 and x = 2 to infinity. The user attempts to change the limits to x = y + 2 to infinity and y = 0 to infinity but finds these options are not available. Another participant agrees that the user's approach appears correct, noting the question's ambiguity regarding the order of integration. The conversation highlights confusion over the limits and the need for clarity in the problem statement.
jaus tail
Messages
613
Reaction score
48

Homework Statement


upload_2018-1-5_16-38-20.png


Homework Equations


I've drawn graph below.

The Attempt at a Solution


Currently their limits are y = 0 to y = x - 2 vertical arrow
and x = 2 to x = infinite. horizontal arrow
upload_2018-1-5_16-38-2.png

Changing limits i get right side graph.
x = y + 2 to x = infinite horizontal arrow
y = 0 to y = infinite vertical arrow
But that is not even there in options. Am i wrong somewhere?
 

Attachments

  • upload_2018-1-5_16-32-54.png
    upload_2018-1-5_16-32-54.png
    13.7 KB · Views: 556
  • upload_2018-1-5_16-38-2.png
    upload_2018-1-5_16-38-2.png
    15.9 KB · Views: 297
  • upload_2018-1-5_16-38-20.png
    upload_2018-1-5_16-38-20.png
    10.6 KB · Views: 828
Physics news on Phys.org
jaus tail said:
Changing limits i get right side graph.
x = y + 2 to x = infinite horizontal arrow
y = 0 to y = infinite vertical arrow
But that is not even there in options. Am i wrong somewhere?

What you have done looks right to me. The question is also odd in that they haven't changed the order of ##dxdy##.
 
  • Like
Likes jaus tail
Thanks.
 
Question: A clock's minute hand has length 4 and its hour hand has length 3. What is the distance between the tips at the moment when it is increasing most rapidly?(Putnam Exam Question) Answer: Making assumption that both the hands moves at constant angular velocities, the answer is ## \sqrt{7} .## But don't you think this assumption is somewhat doubtful and wrong?

Similar threads

Replies
2
Views
2K
  • · Replies 3 ·
Replies
3
Views
1K
  • · Replies 17 ·
Replies
17
Views
4K
Replies
24
Views
3K
  • · Replies 10 ·
Replies
10
Views
2K
Replies
20
Views
2K
  • · Replies 3 ·
Replies
3
Views
2K
Replies
1
Views
1K
  • · Replies 2 ·
Replies
2
Views
2K
Replies
4
Views
3K